Di Replica Last4years Sol

You might also like

Download as pdf or txt
Download as pdf or txt
You are on page 1of 25

SOLUTIONS FOR DATA INTERPRETATION REPLICA QUESTIONS THAT HAVE

APPEARED IN CAT IN THE LAST 4 YEARS


TABLES

108800
= 18133
6
18133 16000
percentage increases =
100
16000
 13%
Choice (3)

Average gross pay =

Solutions for questions 1 to 3:


1.

2.

3.

Let the volume of data transfer in India and Singapore


be 100 units each.
Revenue from data transfer in India = 100 1 = $100
Revenue from data transfer in Singapore = 100 9 = $900
100
Total revenue in India = 100
= $1111
9
100
Total revenue in Singapore = 900
= $4285
21
Total revenue in Singapore is about 4 times that in
India.
Choice (5)

Revenue from data transfer as a percentage of total


revenue for India in 2010 = 27%
Revenue from data transfer as a percentage of total
revenue for Sweden in 2010 = 36%
Let total revenue in India in 2010 be $200 and that in
Sweden be $100
ARDT of Sweden = $6
36
Volume of data transfer in Sweden =
=6
6
Volume of data transfer in India = 6
54
ARDT in India =
=9
6
9 1
The percentage increase =
100 = 800%
1
Choice (3)
It can be seen that if the total revenue received is the
same for the given pairs of countries, only UK and
Spain would have approximately the same volume of
data transfer.
Choice (4)

8.

As after the mutual transfer, the average age of the


Finance department increase by one, it means that the
age of the person who came from the Marketing
department was 20 years older than the age of the
person who was transferred from the Finance
department. Now after the transfer of the employee to
the HR department, as the average age of the
employees left in the Marketing department remained
the same, the age of the employee transferred to the
HR department, was 20 years younger than the
average age, i.e., 36 20 = 16 years.
The new average age of the employees in the HR
department
46 5 + 16 1 246
=
Choice (3)
=
= 41 years
6
6

9.

The new average basic pay of employees in the HR


10,000 5 + 12,000 2 + 16,000 1
department =
8
90000
50,000 + 24,000 + 16,000
=
=
= 11250
8
8
The percentage change = 12.5%
Choice (2)

Solutions for questions 10 to 13:


10. The drink must contain 10% minerals. As there are only
two drinks (A and C) with 10% minerals, the drink can
be prepared in only one way. As A and C have 30%
protein each, they can be mixed to form the drink.
Choice (1)

Solutions for questions 4 to 6:


4.

To get calls from all the colleges, Arun should have


scored at least the highest value of cut-off in each
section, i.e., 44 + 44 + 45 + 44 = 177 and also at least
the highest value of aggregate cut-off for any institute,
i.e., 176.
Choice (2)

5.

The minimum aggregate marks to get calls from two


colleges is 171. If he scores 50 each in three sections
he needs to score at least 21 marks in the fourth
section.
Choice (3)

6.

Four colleges have a cut-off for section C and the


remaining two colleges have a cut-off for section D.
If a student misses the cut-off in these two sections,
he/she would miss calls from all the colleges.
The maximum possible marks such a student gets is
50 + 50 + 40 + 42 = 182.
Choice (3)

Solutions for questions 7 to 9:


7.

The new gross pay of the employee transferred


80
= 16,000 +
16000 = 16,000 + 12,800 = 28,800
100
The gross pay of the current employees in HR
department = 16000 5 = 80000
New gross pay of the six employees = 80,000 + 28,800
= 1,08,800

11. None of the choices (1), (2) and (3) can be used to form
the drink with 10% fat and at least 30% protein. For C
and E to form the drink with 10% fat and at least 30%
protein, if they are mixed in the ratio x : y (say)
x (50 )+ (0 )
= 10, x : y = 1 : 4
x+y

1(200 )+ 4 (100 ) 600


= 120
=
5
5
Similarly the ratio for D and E is 1 : 3 and the cost per
800
unit is
= 200
4
The cost per unit is the least for C and E.
Choice (4)
cost per unit =

12. The drink should have at least 60% carbohydrate.


Further in the mixture formed by B, C and E, the
proportion of B should be maximum and the other two
should be minimum to get the lowest per unit cost.
Among the given options only Choice (2) and (5) satisfy
the condition having 60% carbohydrate and of these,
choice (5) has the lowest per unit cost.
Choice (5)
13. A and B when mixed in equal proportions, the protein
30 + 20
content will be only
= 25%, which is less than
2
required. D and E when mixed in equal proportion, the

5 + 45
= 25% which is
2
less than required. Similarly B and E and C and D when
mixed in equal proportion the combination will have less
than the required percentage of minerals and
carbohydrate respectively. Only A and E when mixed in
equal proportion would yield a mixture with all the
contents in the required amount.
Choice (5)

carbohydrate content will be only

Solutions for questions 14 to 17:


14. If one observes the values given for the different
parameters, the values that were varying with
production, i.e., value was increasing when production
increased and value decreasing when production
decreased are material, labour and operating cost of
machines. All the remaining costs, i.e., rent of building,
consumables, rates and taxes, repair and maintenance
expense and selling and marketing expenses are fixed.
Hence, there will be no change in these costs. The total
fixed cost = 1800 + 600 + 1200 + 8700 + 2100 = 14400
The cost/unit for different variable costs is as follows.
Material = `50 per unit.
Labour = `20 per unit
Operating cost of machine = `30 per unit
Total = `100 per unit
Selling price per unit = `125 per unit
14400
Total cost/unit for 2100 units is `100 +
2100
= `107
Choice (2)
15. For one product,
Selling price = `125
Variable cost = `100
_________________
Difference = `25
_________________
Now, to avoid loss, the company has to offset the fixed
cost (i.e., 14400) for which it has to produce a total of
14400
= 576 units.
Choice (3)
25
16. The reduction in selling price per unit = 5% of 125
= 6.25
New selling price = 118.75
Total fixed costs = `14400
Variable cost per unit = `100
Now the total profit increases with the increase in
number of units sold and the maximum profit is
obtained when the company sells and 3000 units.
Choice (5)
17. The given condition is that if the company sells upto
2100 units, the selling price per unit is `125 and if the
company sells 2550 units, the selling price per unit for
all the units is `120. The profit of the company
increases upto a production figure of 2100 units, from
the 2100th unit to the 2101st unit, the total profit
decreases drastically and from the 2101st unit to the
2550th unit, the profit again increases.

Hence, the profit would be maximum at the production


figure of 2100 units or at 2550 units.
Production
Selling price / unit (s)
Variable cost / unit (v)
SV
(S V) production
Total fixed cost
Total profit

2100 units
`125
`100
`25
25 2100
= 52500
14400
38100

The maximum profit is `38100

2550 units
`120
`100
`20
20 2550
= 51000
14400
36600

Solutions for questions 18 to 21:


18. The costs of a refrigerator, an air conditioner and a
music system in different countries are.

(00 U.S. dollars)

Refrigerator
Air conditioner
Music system
Total

India Thailand Malaysia Singapore USA


11 + 5 13 + 5
11 + 6
13 + 4
20
9+7
12 + 5
10 + 8
12 + 5
23
8.5 + 9 10 + 6
8+4
13 + 4
20
49.5
51
47
51
63

The cheapest is in Malaysia.

Choice (3)

19. As given in the previous question, the total cost will be


highest in India (850 + 900 = 1750)
Choice (1)
20. Cost in India = 300 + 500 = 800
Cost in Thailand =450 + 600 = 1050
Difference = 250 32.9 = 8225
Duty = 1500
Required difference = 6725

Choice (4)

21. Cost in India with dollar at `40.92 = 550 40.92


2500
22500
Cost in India with dollar at 35 =
= 650
35
Cost in Singapore = 900
Required difference = 250
Choice (2)
Solutions for questions 22 to 26:
22. Let us check the possible short routes from A to J.

Total cost

Total distance

`1470

1430 km

Rs. 625
Rs.1225
D
J
Rs. 25 km
825 km

`1850

1250 km

Rs. 850
Rs. 575
F
J
670 km
485 km

`1425

1155 km

Rs.1225
Rs. 445
G
J
675 km
485 km

`1670

1090 km

Rs. 925
Rs. 210
H
J
975 km
200 km

`1135

1175 km

Rs. 675
Rs. 215
C
395 km
205 km

`1465

1085 km

Rs. 335
Rs.1135
A
B
J
280
1150 km

Rs. 575
J
485 km

The shortest possible route is A C F J.


The cost is `1465.
Choice (4)
23. The route with the least cost is A H J, with a total
cost of `1135. As the cost of the new service is 5% less
then `1135, it should be 1135 (5% of 1135) = 1078.
Choice (2)
24. If C, D and H are closed, then the minimum cost of
Choice (3)
travel is for A F J, i.e., `1425.
Pr ice
to be as minimum as possible.
dis tan ce
It is less than 1 in only the cases A H, B J and

25. We want the

Choice (1)

C D. Considering the cases involving the above


routes.
Route

Price / Distance

AHJ

1135
1175

Taking margin of 10%


into Account
1135
10

1175
11

ABJ

1470
1430

1470 10

1430
11

It will be the least for A H J and is


=

103.2
= .88
117.5

27. Percentage of male employees in the production


288
100 = 45
Choice (2)
department =
640
28. Post graduates in the marketing department = 32
25
32 = 8
Male postgraduates =
100
Female post graduates = 32 8 = 24
Male non post graduates = 48 8 = 40
Required difference = 40 24 = 16
Choice (5)
29. Percentage of male post graduates in the marketing
32
department =
100 = 40
Choice (1)
80

1135 10

1175 11

Choice (2)

26. The cost / kilometer is the least for A H J and the


distance is 1175 km.
Choice (4)
Solutions for questions 27 to 30:

30. The number of male post graduates in the production


department = 144.
Female post graduates = 352 144 = 208
The number of male and female post graduates and
male and female employees who are not post
graduates are as follows.

With the given information we can deduce the number of males


and postgraduates in the different departments as follows:
Department
Marketing
Accounts
Production
Total

Total
80
80
640
800

Male
48
44
288
380

Post graduates
32
40
352
424

Post graduate

Non Post graduates

Male

Females

Males

Females

144

208

144

144

It can be seen that except female post graduates all


other groups (male posts graduates, male and female
non post graduates) have the same number of
employees.
Choice (3)

Solutions for questions 31 to 33:


31. The total number of bookings made is the highest in Q3 and so the average number of bookings per month is also the
highest.
Choice (C)

Month
Number of bookings

Jan
346

Feb
412

Number of deliveries

200

March
380
216
(146)

April
450
160
(196)

May
308
225
(220)

32. The values shown in the brackets are of the booking


made 2 months ago.
Number of deliveries made in August from the bookings
made in June = 200.
Number of deliveries made in December from the
bookings made in November = 278
278
Choice (A)
= 1.39.
200
33. We only need to check the revenue for quarters Q3 and
Q4.
Revenue (in `) from Q3 = (462 + 333 + 345) 43,100
= 4,91,34,000
Revenue (in `) from Q4 = (250 + 506 + 370) 44,000
= 4,95,44,000
the highest revenue is obtained from Q4 i.e.,
`4,95,44,000
Choice (B)
Solutions for questions 34 to 36:
34. The given condition occurs in the case where the
number of wins is in the range 16 18. Choice (B)
rd

35. From the given table, the 3 least percentage occurs in the
99 94
100
100 =
last row i.e., for 3133, which is
95
19
5
Choice (C)
= 5 %
9

June
359
170
(225)

July
462
159
(138)

Aug
333
296
(200)

Sep
345
134
(166)

Oct
250
50
(199)

Nov
506
125
(295)

Dec
370
278
(125)

36. The given condition is satisfied in the case, where the


number of wins is in the range 25 27 i.e.,
68 + 64 + 63
Choice (A)
= 65 = 2.5 26
3
Solutions for questions 37 to 39:
37. Investment (in `) in NLP Industries before withdrawal
= 12.5% (12,00,000) = 1,50,000
Investment (in `) in NLP Industries after withdrawal
= 16% (9,00,000) = 1,44,000
the percentage change in investment
1,50,000 1,44,000
=
Choice (D)
100 = 4%
1,50,000
38. The return on investment for Mr. Anil
2
25
=

7,00,000 = 3500
100 100
The return on investment for Ms. Shivani
2.5 10
=
13,00,000 = 3250

100 100
Therefore the required difference = (3500 3250)
Choice (A)
= `250

39. The three persons A, B, and C made an investment of


`10 lakh, `20 lakh and `21 lakh respectively such that
their investments fall under the schemes X, Y, Z
respectively.

Their combined return on investment (in ` )


= 2% (10,00,000) + 2.5% (20,00,000) + 3% (21,00,000)
= 20,000 + 50,000 + 63,000 = 1,33,000
Their combined return on investment after the firm
increased the rate of return
= 2.2% (10,00,000) + 3% (20,00,000) + 3.3% (21,00,000)
= 22,000 + 60,000 + 69,300 = 1,51,300
The required increase (in `)
= 1,51,300 1,33,000 = 18,300
Choice (D)

The franchise in Bengaluru will earn more revenue then


the establishment fees (in each of the two centres) after
one year.
Choice (D)
44. If a customer spends on an average `300 and `130 at a
Foodie restaurant in class A center and class B center
respectively, then the total number of customers who
are required to come such the revenues are not less
than the
establishment fees would be the i.e.,
132 10 5
104 10 5
+
= 44,000 + 80,000 = 1,24,000
300
130
Choice (B)

Solutions for questions 40 and 41:


40. Percentage contribution of mono speaker of the
1000
100 = 12.3%
company NOSY in 2001 =
8100
Percentage contribution of mono speaker of the
1600
100 = 17%
company BOSS in 2001 =
9400
Percentage contribution of mono speaker of the
companies NOSY and BOSS in 2003 are 13.5% and
19.6% respectively.
the percentage contribution of mono speakers of
both the companies increased.
Proceeding, similarly we observe that for no other type
of music systems of both the companies, the
percentage contribution increases.
Choice (B)
41.

Type of music system

Mono speaker
Dual speaker 1000w
Dual speaker 2000w
Four speakers 5000w
Home theatre

Percentage
contribution
in 2001
12.3%
22.22%
28.4%
17.3%
19.75%

Percentage
contribution
in 2003
13.54%
22.9%
18.75%
23.95%
20.8

The maximum change in percentage points occurs for


Dual speaker 2000W.
Choice (C)

42. The total number of cars sold by showroom A and


showroom B at the end of 7 days are 209 and 221
respectively.
209
=
100 94.5%. Hence statement is true.
221
The total number of cars sold by showrooms on odd
numbered days = 16 + 35 + 33 + 51 + 60 = 195
The total number of cars sold by showroom B on even
numbered days = 19 + 42 + 29 + 52 + 81 = 223.
90%(223) = 200.7
Statement is also true.
Choice (C)
Solutions for questions 43 and 44:
126 10 5
= 52,500 > 51,860;
240

75 10 5
< 42,500
180
a restaurant in class B center but not class A center
will earn more revenue then the establishment fee in
one year.

Bengaluru:

144 10
240

45. As no information is given regarding the percentage of


dropouts for districts R and S in few years, (A) cannot
be definitely a false statement. As no information is
given about the number of enrolments in each districts
in any of the years, statements (B) and (D) cannot be
confirmed.
(C) is definitely false because the dropout percentage of
district Q in any of the given years is greater than that of
each of the other districts. So, the overall dropout
percentage would also be the highest.
Choice (C)
46.

District
P
Q
R
S
T

Minimum number of achievements


5
1
2
0
2

total number of achievements (minimum = 5 + 1 + 2


+ 0 + 2 = 10)
Choice (B)
Solutions for questions 47 and 48:
47. The rise in temperature (in C) per hour =

Solution for question 42:

43. Hyderabad:

Solutions for questions 45 and 46:

= 60,000 < 60,200

90 10 5
= 50,000 < 50,246
180

46 25
=7
7

temperature (in C) in city Q at 10 a.m.


= 3 (10 5) + 25 = 40
Choice (D)
48. Temperature in city P at 3.30 p.m.
1
42 29
= 42
3
6
2

= 42

91
34.5C
12

Similarly,

City
Q
R
S
T
U

Temperature at 3.30 p.m.


35.5 C
37 C
38.5 C
36.66 C
33.5 C

city S has the highest temperature at 3.30 p.m.


Choice (C)
Solutions for questions 49 to 52:

P
Q
R
S
T
U
Traffic flowing from 3,346 3,752 2,536 2,620 2,952 3,060
Traffic flowing to 3,504 2,612 3,308 2,852 3,050 2,940
49. Total traffic through the route P Q = 964 + 846 = 1810.
Similarly verifying it is easy to see that the maximum
traffic flow occurs through the road connecting PQ.
Choice (A)

50. Looking at the table and relating the diagonal elements,


it is easy to see that the 2nd least traffic flow occurs
through the road connecting Q S.
Choice (D)
51. From the above table, traffic flowing from city Q is the
greatest i.e., 3752 vehicles.
Choice (B)
52. From the above table, the difference in traffic flow is the
least for city T i.e., 3050 2952 = 98.
Choice (C)
Solutions for questions 53 and 54:
53. Interest amount for Mr. A (in `) = 3,600 2.2 = 7,920
Interest amount for Mr. B (in `) = 3,800 3.6 = 13,680
The required difference (in `) = 13,680 7,920
= 5,760
Choice (B)

1
[5.6 4,800 + 6.4 4,000]
2
1
=
[26,880 + 25,600]
2
Choice (C)
= `26,240.

54. The required average =

Solutions for questions 55 and 56:


55. Let the total number of employees in company X,
company Y and company Z be x, y and z respectively.
Male employees in company x who owns both four
wheeler and two wheeler = 0.7x 0.15 (Q 45 + 65 + 5
100 = 15) = 105x
Female employees in company y who owns both four
wheeler and two wheeler = 0.3x 0.1 = 0.03x.
The total number of employees in company X who
owns both four wheeler and two wheeler
= (0.105 + 0.03)x = 0.135x
percentage of employees = 13.5%
Similarly for company Y, the required percentage is 26%
Similarly for company Z, the required percentage is 6%.
Choice (B)
56. Let the number of employees in either of the companies
be n.
The number of male employees in companies Y who satisfy
1
[100 (30 + 20)]n = 0.5n
the given condition =
100
similarly the required number of employees in company

Z=

1
[100 (20 + 10)]n = 0.7n.
100

the required percentage =

No. of Bikes Sold

RL-100
BCZ
Thunder
WB-150
Muzzle

19,500
37,500
30,000
45,000
18,000

Average selling
price (in `) 2007
40,000
25,000
31,000
20,000
52,000

2008

45,000
28,000
35,000
23,000
55,000

57. From the above table, the percentage increase in the


average selling price is the highest for WB-150.
23,000 20,000
100 = 15%
Choice (C)
20,000
58. The required average (in `)
40 + 25 + 31 + 20 + 52
=
103
5
168
=
10 3 = 33,600
5

59. The gain from the shares of company IV in 2006 was =


[ 448 + 432 2( 456)]
132 +
2
= 132 16 = 116
Choice (1)
60. We can only evaluate the return from the shares of
company III in the years 2002 to 2009. The returns
were as follows:

Year
Gain

2002 2003 2004 2005 2006 2007 2008 2009


122.5 158.5 155 150 170 148 172 195

The highest percentage increase is from 2002 to 2003


and it is 29.38%
Choice (2)
Solutions for question 61:
61. To find the median, arrange the per capita incomes in
descending (or ascending) or order.

Per capita income ($)


24,369
24,337
23,484
19,207
15,350
13,746
13,477
11,692
10,372
5,663
4,965
3,523
2,916

Country
Switzerland
Germany
United states
United kingdom
New Zealand
Swedes
France
Spain
Hong Kong
Brazil
Latherier
Mexico
Romaine

Out of the 13 countries, the median is the country


placed 7th.
France with a per -capita income of $13,477.
40% of 13,477 = $ 5390.8.
There are 10 whose per capita income is more than
$5390.8.
Choice (2)
Solutions for question 62:

0. 5 n + 0. 7 n
100 = 60%.
2n
Choice (D)

Solutions for questions 57 and 58:


Model

Solutions for questions 59 and 60:

62. Given, intra-state services accounted for 60% of total


revenues.
2880
Total revenues =
= `4800 crore.
0.60
Total revenues from non A/C general category in
intra-services is given to be 50% of revenues from intrastate services.
Revenues from non A/C general category in intrastate services = 50% of 2880
= `1440 crore
Choice (2)
Solutions for question 63:
63. The total production of the top four coal producing
countries is 2536.7 + 1039.2 + 478.2 + 393.9 = 4448 mt
The total production of the bottom four coal producing
countries is 76.7 + 76.6 + 145.8 + 174.9 = 474 mt.
474
The required percentage =
100 = 10.66%
4448
Choice (1)

Choice (B)

Solutions for questions 64 and 65:


64. Male students who were eligible for selection were A, F,
G and N and the female students who were eligible for
selection were L and O
Therefore the required ratio is 2 : 1
Choice (1)
65. x = 6 and y = 4
There fore 2x = 3y is in the correct choice. Choice (4)
Solutions for question 66 and 67:

The Total costs, Operating Expenses, Revenue and the


Profitability of the company in the five years are given in the
following table.
Year
2009
2008
2007
2006
2005

Total cost (in


`)
92200
83700
89600
96600
104000

Operating
Revenue Profitability
Expense (in `)
(in `)
(in `)
24050
104200
0.2308
21775
96600
0.2254
22000
112400
0.1957
24730
128200
0.1929
26580
130600
0.2035

66. The profitability of the company was the least in the


year 2006
Choice (2)
67. With respect to the previous year in the years 2006,
2007 and 2008 were 7.11%, 7.24% and 6.58%
respectively. In 2009 the total cost increased when
compared to 2008. Therefore the maximum decrease
was in 2007 and it was 7.24%.
Choice (2)
Solutions for questions 68 and 69:

Sections
A
B
C
D
E
Total

Score
Score less Score from
Total no of
greater than
than 45
45 to 85
students
85
28
72
24
124
15
68
36
119
18
52
28
98
29
58
47
134
30
60
35
125
120
310
170
600

68. Percentage of the total number of students getting


120
scores less than 45 =
100 = 20% Choice (4)
600
69. For the sections A, B, C, D and E, the maximum
number of students getting 48 or more in the
examination was 96, 104, 80, 105 and 95 respectively.
Thus the highest among the above values is 105.
Therefore the maximum number of students from a
section who passed in the examination was 105.
Choice (3)
Solutions for questions 70 to 72:
70. The growth in exports of the 4 companies from 2003 04 to 2004 - 05 are as follows..
3.15
Rahual & co:
100 = 25.9%
12.15
2 .1
Chandu & co:
100 = 14.9%
14.1
2 .8
Shiva & co:
100 = 30.1%
9 .3
3.32
Kanta & co:
100 = 31.9%
10.41
Hence Kanta & co has the highest growth in exports.
Choice (4)

71. The growth rate in imports of the 4 companies


from 2002 03 to 2003 04 are as follows:
6.53
Rahual & co:
100 = 127%
5.14
4 .7
Chandu & co:
100 = 40.5%
11.61
0.67
Shiva & co:
100 = 7.7%
8.72
4.05
Kanta & co:
100 = 54.3%
7.46
Hence the growth rate of imports is the least for
Shiva & co.
Choice (3)
72. Trade deficit = imports exports
The trade deficit of the 4 companies in 2004 05 are as
follows
Rahul & co: 2.11
Shiva & co: 0.9
Chandu & co: 1.07
Kanta & co: 0.6
Rahul and co has the highest trade deficit in 2004 2005
Choice (1)
Solutions for questions 73 and 74:
73. The number of employees who did not cross the cut off
for all the 5 companies are as follows.

A
B
C
D
E

No. of employees who did not cross the cut off.


120
225
100
200
275

Hence E has rejected the maximum number of


employees.
Choice (4)
74. The number of employees who got more than 90% for
the 5 companies are as follows:

A
B
C
D
E
Total

Greater than
90% marks
30
36
30
96
115
275

The required percentage is

Cut off cleared


180
225
150
400
300
1255

307
= 24.5% 24%
1255
Choice (3)

75. The sales of a company = The no .of units produced


the closing stock.
The sales of all the 5 companies in 2009 are as follows

Company
P
Q
R
S
T

Sales
10515
14310
9225
7755
11135

Hence S had the least sales in 2009.

Choice (2)

76. The sales of R in 2008 = 9000 675 = 8325


The sales of R in 2009 = 10000 775 = 9225
R had lower sales in 2008.
Choice (1)

Solutions for question 77 and 78:

Solutions for questions 84 and 85:

77.

84. The yield return of R in the years are as follows:

Expenses +
Overhead
(In `)
12000
13500
17750
16750
19000
17450
21375
923175

Family:

Total income
(in `)

Savings
(in `)

Kapoor
Khanna
Kirsten
Kumble
Khan
Kittu
Kala

147000
105000
168000
140000
165000
120000
196000

135000
91500
150250
123250
146000
102550
174625

The total savings made by all the families was `923175


Choice (2)

Hence the highest yield return is in 2021


Choice (1)
85. The yield return for Q in the years are as follows

2004
2005
2006
2007
2008

Solution for question 79:

Hence the highest percentage increase is in 2008


Choice (4)
Choice (3)

Solutions for 80 and 81:


80. The difference in the number of students studying in
government schools in all the states in 2008 are as
follows.

AP
MP
UP
Karnataka
Kerala
Tamil Nadu

Difference
1800
1600
1300
1400
2200
5200

Solutions for questions 86 and 87:


86. From the table we can easily observe that the average
marks are the highest for , X and X.
Hence these classes would satisfy the statement
"the higher the average marks, the higher are the
number of students".
Choice (3)
87. The statement the lower the number of students, the
higher the average marks can be verified through the
options.
Classes and have higher number of students, hence
they do not satisfy the statement.
The correct choice is (A)
Choice (1)
Solutions for question 88:

The maximum increase is for Tamil Nadu,


Choice (4)

88.

81. We can observe that the state which has consistent


increase in the number of students from 2007 to 2009 is
UP.
Choice (2)

82. The median of the total number of students is


18 + 17
=17.5
2
Hence A, C and F have more number of students than
the median.
Choice (2)
83. The number of failed students in each section is given
the table.
Hence the number of failed students is the highest more
in section C

No. of students failed


6
8
12
7
3
10

The number of students this year in the 6 states


Number of students
13,21,000
17,46,000
13,90,000
19,14,000
12,88,000
10,88,000

AP
UP
MP
Bihar
Assam
Orissa

Solutions for questions 82 and 83:

Section
A
B
C
D
E
F

Yield Return
1676.5
2204.5
2633.3
2000
2720
4105.3

2003

78. The increase in income of the Khan family is `3300


The decrease in expenses is `570
The increase in saving is 3300 + 570 = `3870
Choice (3)

79. The healthy drinks are S are X


The other drinks are unhealthy.
Hence the required ratio is 2 : 8 = 1 : 4

Yield Return
1557.14
1574.1
1884.6
2021.7
1595.2
1525

2003
2004
2005
2006
2007
2008

Hence MP has the third highest number of students


this year
Choice (2)
Solutions for question 89:
89.

Given Lakshmi spends 20% of the revenue earned


from each investment to maintain her house. So let us
calculate the revenue for each business in which she
invested.

Investment
Revenue

X
16.2
16.96

Y
14.5
14.72

Z
12.9
13.2

Hence the maximum profit is obtained from X


Choice (1)

Choice (3)

Solutions for questions 90 and 91:

90. The income and expenditure for the four regions in


2007 are as follows.

Income

Expenditure

Ratio

North

33.8

34.5

0.98

South

33.8

35.2

0.96

East

31.9

32.7

0.975

West

40.1

41.3

0.971

5.

100
4800 = 12,000
40
100
4200 = 10,000
Number of bears in the world =
42
The number of deers and wild bisons in South America
are 6,000 (25% of 24,000) and 5,400 (30% of 18,000)
respectively.
Number of wolves in South America
= 25,800 (4,800 + 6,000 + 5,400 + 4,200) = 5,400
total number of wolves in the Amazon forest = 75%
of 5,400 = 4,050.
Choice (B)
Number of pythons in the world =

6.

Species
Pythons
Deers
Wild Bisons
Wolves
Bears

BAR GRAPH
Solutions for questions 1 to 4:
1.

2.

The percentage growth rate in 2007 over 2006


250 190
=
100 = 31.5%
190
Had the percentage growth from 2007 to 2008 been
31.5%, the estimated revenue would have been
131.5
250
= 329
100
The required difference 329 305 = 25 (approximately)
Choice (1)

Solutions for questions 7 to 9:


7.

Year
2003 2004 2005 2006 2007 2008 2009 2010
Gap in
150 170 150 140 110 85 60 50
million USD
Absolute
percentage
13 12
6
21 22 30 17
change
The absolute value of the percentage change in the
growth rate was the highest in 2008-09. Choice (4)
4.

The growth rate in 2005 (over 2004)


135 90
=
100 = 50%
90
The growth rate in 2007 (over 2006)
250 190
=
= 32%
190
The required percentage

The percentage increase is maximum in case of


company R i.e.,

5400 4480
100 20.5%
4480

Choice (C)
8.

Since the cost of PC is same for all the companies


market share of Q in 2009
4200
=
21.76%
5600 + 4200 + 5000 + 4500
Market share of Q in 2014
110% ( 4200 )
100 = 28.37%
=
110% (5600 + 4200 + 500 )
the difference in percentage points = 28.37 21.76
= 6.61.
Choice (A)

9.

Looking at the options it is enough if we check for the


market share of S in 2006 and 2007.
Let the price per PC of A, B, C and D `x, `2x, `x and
`2x respectively.
Market share of S in 2006
2 5800
100
=
4200 + 3000 2 + 448 + 5800 2)

Year 2003 2004 2005 2006 2007 2008 2009 2010


Men
60
63 66.15 69.5 73 76.65 80.5 84.5
Women 40
44 48.4 53.25 58.5 64.5 71
78
Total
100
162.5

3.

Number in Amazon Forest


80% (4,800) = 3,840
70% (6,000) = 4,200
80% (5,400) = 4,320
75% (5,400) = 4,050
95% (4,200) = 3,990
Choice (B)

Let the number of people who used the company's


products in Asia in 2003 be 100.
The number of men and women who used the product
in the different years are

the approximate percentage growth = 62


Choice (1)
The percentage change in the gap between the revenues
from the US and Asia in the different years are

Choice (3)

Solutions for questions 5 and 6:

Hence the required ratio is the highest for the North


region.
Choice (1)
91. The states in which the per capita income increased by
more than 5% are J and K, West Bengal, Gujarat and
Maharashtra
In the remaining states the per capita income did not
increase by more than 5%
Hence the required ratio is 1 : 1
Choice (2)

50 32
100 = 35 (approximately)
50

116
100 44%
42 + 60 + 44.8 + 116
Market share of S in 2007
108
54 2
=
100 =
100
43.5 + 28 2 + 48.5 + 54 2
256
= 42.2%
Choice (A)
=

10. Let us consider the selling prices of the four models in


2004 as 3k, 4k, 5k and 6k respectively.
Selling prices in the years.

Models
P
Q
R
S

2004
3k
4k
5k
6k

2005
4.5k
6kl
7.5k
9k

2006
6k
8k
10k
12k

Sales revenue of Q in 2004 = 750 (4k) = 3000k


Sales revenue of R in 2006 = 500(10k) = 5000k

Therefore the sales revenue of Q in 2004 by


2
5000k 3000k
Choice (3)
100 = 66 %
3
3000k

Solutions for questions 11 to 13:

Car

2007
Production

2008
Sales

Production

Sales

Production

Alto

13000

8000

15000

10000

14000

9000

22000

20000

21000

18000

25000

22000

Estio

20000

18000

21000

20000

22000

16000

4.

Section
QA
LR

Choice (3)

VA
RC
DI

Choice (1)
13. The exports of Swift in
2007 = 2000
2008 = 3000
2009 = 3000
The ratio of exports to sales in
2007 = 0.1
2008 = 0.17
2009 = 0.14
The required ratio is the highest in 2008

6.

7.

Solutions for questions 1 to 6:

QA

LR

VA

RC

DI

Total

51
105
120
80
84

69
30.8
45
64
42

60
35
60
32
91

45
56
54
64
70

75
53.2
81
80
63

300
280
360
320
350

VA
50

The marks obtained by the student in the RC section is


the highest in AIMCAT 5
Choice (B)

Solutions for question 7:

PIE CHART

LR
80

Percentage change
56 34
100 > 50%
34
55.2 33.6
100 40%
55.2
36.4 24
100 > 50%
24
56 36
100 > 50%
36
62.5 52.4
100 16%
62.5

Choice (D)

Choice (2)

QA
100

The student showed the highest percentage increase in


84 51
Choice (B)
QA section i.e.,
100 = 64.7%
51

5.

12. The ratio of production to sales of Alto in


2007 = 1.625
2008 = 1.5
2009 = 1.55
The required ratio is the highest in 2007

Maximum actual score

Sales

Swift

11. The total production of all three Cars in


2007 = 55000
2008 = 57000
2009 = 61000
The total sales of all three Cars in
2007 = 46000
2008 = 48000
2009 = 47000
The required ratio in
2007 = 1.19
2008 = 1.18
2009 = 1.29
Hence the ratio is the highest in 2009

Section
Exam
AIMCAT 1
AIMCAT 2
AIMCAT 3
AIMCAT 4
AIMCAT 5

2009

RC
60

DI
100

60
100 = 120%
50
Choice (D)

1.

The required percentages =

2.

Maximum possible actual score in RC section = 60.


The least difference occurs in the AIMCATs 2 and 4 i.e.,
60 56 = 64 60 = 4
Choice (A)

3.

From the above tables, the given condition is satisfied


in AIMCAT 3 and AIMCAT 5.
Choice (B)

Given the total number of students = 1000


Dancing = 45% = 450
Embroidery Classes = 5% = 50
Singing = 20% = 200
Karate = 15% = 150
Painting = 15% = 150
Now only Boys chose Karate. Hence a total 150
students in Karate are all boys.
Only girls chose Embroidery classes. Hence a total 50
students in Embroidery are all girls.
Also 80% of students in Singing are girls.
Hence 160 students in Singing are girls and 40 are
boys.
Similarly 80% of students in Dancing are boys.
Hence 360 are boys and 90 are girls, who are in
dancing
In Painting the ratio of boys to girls is 1 : 1
Let us tabulate the data.

Dancing
Singing
Painting
Karate
Embroidery Class
Total

Boys
360
40
75
150
0
625

Girls
90
160
75
0
50
375

Total
450
200
150
150
50
1000

If Painting & Singing are mixed then the ratio of boys to


girls is 115 : 235 = 23 : 47
Choice (3)

LINE GRAPH

Solutions for question 8:


8.

Given the ratio of the number of employees in central to


state government jobs is 6 : 1.
Let the central government jobs be 600 and the state
government jobs be 100.
The number of central government employees in A.P =
150.
The number of state government employees in Kerala =
25.
The required ratio is 150 : 25 = 6 : 1
Choice (1)

Solutions for questions 1 and 2:


1.

Profit on a normal day = 7000 6500 = `500


Profit when 300 units are sold = 10,500 9000 = `1500
1500 500
Required percentage =
100 = 200%
500
Choice (C)

2.

Cost when 200 units are produced = `6500


Cost when 350 units are produced `10,000

Solutions for 9 and 10:


9.

Additional cost /unit =

The number of employees in each department of P


and Q.

HR
Academic
Operations

P
1750
10500
5250

Q
3780
3060
11160

Total number of employees in both companies in


the HR department = 5530
5530
= 15.5%
Hence the required percentage =
35500
Choice (2)
10. The number of Academic employees = 13560
The number of Management department employees
= 21940
13560
The required ratio =
= 0.618 0.62
21940
Choice (2)

Solutions for questions 3 and 4:

The energy consumption of Geyser in a week is 7 kWh and


we know the family uses the Geyser for 2 hrs in a day. Hence
for 14 hrs in a week the energy consumption is 7 kWh.
Hence the energy consumption of a Geyser per day is 1 kWh.
Now, energy consumption of Refrigerator in a week is
14 kWh and the family uses Refrigerator throughout the day.
Hence, the energy consumption of Refrigeration per day is
2 kWh.
Similarly the energy consumption for TV in a day is 2 kWh.
The energy consumption for Washing machine in a day is
1
4
kWh and for Grinder is
kWh
7
7
3.

(a) Energy consumed by TV for 3 days = 6 kWh.


Energy consumed by Refrigerator for 3 days
= 6 kWh.
Hence Choice (1) is false.
(b) Energy consumed by Geyser for 4 days = 4 kWh.
Energy consumed by Grinder for 7 days = 1 kWh.
Hence Choice (2) is false.
(c) Energy consumed by Washing Machine in a week
= 4 kWh.
Energy consumed by Geyser for 2 weeks = 14 kWh.
Hence Choice (3) is true.
(d) Energy consumed by TV for 2 days = 4 kWh.
Energy consumed by Washing machine for one
week = 4 kWh
Hence (d) is false
Choice (3)

4.

The fixed cost increased by 25 %. Hence the new fixed


1
cost is `60 +
(60) = `75.
4
Hence the increment in the total cost is
15
15
100 =
100 = 12.5%
4
120
60 + (0.35 30)
7
Choice (2)

Pie Charts + Bar Charts


Solutions for questions 1 and 2:

Number of students in each discipline is as follows:


Number of students
Discipline
Marketing
Finance
Operations
Systems
HR

Number of students
3780
1260
1008
1512
252

Number of males and females in each discipline are as


follows:

Marketing
Finance
HR
Systems
Operations
Total

Males
2079
819
1008
840
576
5322

Females
1701
441
1512
672
672
4758

Difference
378
378
504
168
144

1.

The total number of female students in the institute was


less than the total number of male students by
5322 4758
100 = 10.6%
Choice (3)
5322

2.

The difference between the number of male and female


students was the highest for HR.
Choice (1)

10,000 6500 3500


=
= `23.
150
150
Choice (B)

DATA SUFFICIENCY
Solutions for questions 1 to 4:
1.

From A, as 60% of the newly joined employees were


not managers, the remaining 40% of the newly joined
employees were managers. It is given that
10 managers had newly joined.
40% = 10 100% = 25
Hence, A alone is sufficient.
B gives no data, it is just an assumption. Choice (1)

2.

From A and the given condition, either Babu or David


got the highest rank.
Hence, A alone is not sufficient.

10

From B and the given condition, either David or Amar


can be the highest ranker. Hence, B alone is not
sufficient.
Combining A and B, David must get the highest rank.
Choice (4)
3.

4.

It is given that, 30% of the students are boys, which


implies 70% of the students are girls. Also 10% of the
girls are athletes. 10% (70%) = 7% of the students
are female athletes.
From A, 25% of the students are athletes
Hence 25 7 = 18% of the students are boys who are
athletes. So, A alone is sufficient.
From B,
Number of boys who are athletes = 120% of the girls
who are athletes.
As 7% of the students are girls who are athletes, 120%
(7%) = 8.4% of the students are boys who are athletes.
So, B alone is also sufficient.
Choice (3)
Clearly, A alone is not sufficient, as we do not know
how many points the opponent scored.
B alone is also not sufficient, as we do not know how
many points team A scored.
Combing A and B,
If the score at the half time was say 0-25, then the
match would have ended in a tie at 35-35. So, team A
did not win. Had the score at half time been, say, 10-35,
then in the end it would have been 45-35 and team A
would have won. So, we cannot answer the question
even after combining both the statements.
Choice (5)

Again yz = 21 the different possibilities are


1 21
37
But we do not know if x , y, z are natural numbers or
not. For eg if y = 9, we can get.
7
xy = 18 as 29 and yz = 21 as 9
3
There will be infinite possibilities like this, so statement
A is not sufficient.
Statement B above is also not sufficient as it gives
information regarding x and z only and nothing about y.
Combining both the statements, we can conclude that
x = 6, y = 3 and z = 7. Thus z is the maximum.
2.

Using statement A alone we have SEVEN = 19 and


FIVE = 14, without knowing anything about the values
of individual alphabets, we cannot answer the question.
Using both the statements together we can conclude
that 2 (N) + I + E = 7
N = 1 and I and E are 2and 3 or 3 and 2
So F + V = 14 5 9. F and V could be 4, 5 or 5,4.
SEVEN = 19
If E = 2 and V = 5, we get S = 9 whereas if E 3 and
V = 5, we get S = 7.
So we cannot determine S uniquely.
Thus the question cannot be answered even by
combining both the statements.

3.

The number of days that Raju's dad goes to the Shiva


365
temple in a year is
= 121 days.
3
The number of days that Raju's dad goes to the
365
Venkateshwara temple is
= 91 days.
4

PPL
1.

Let the cost of the new car be `x.


2
x.
5
From statement , we cannot answer the question as
neither the cost of the new car nor the cost of the old
car is given.
Statement alone is not sufficient.
From statement , we only know regarding his personal
saving but nothing about the cost of the cars.
Statement alone is not sufficient.
By combining both the statements, we have the
following information.
2
Amount borrowed from his friend = 60% x
5

The number of days that Raju's dad goes to the

Therefore the cost of the old car = 40% (x) =

365

Saibaba temple is
= 52 days.
7
The number of days that Raju's dad goes to both the

365

Shiva and the Venkateshwara temple is

12
= 30 days.
The number of days that Raju's dad goes to both the

365

Venkateshwara and the Saibaba temple is

28
= 13 days.
The number of days that Raju's dad goes to both the

6
32
x
x =
5 5 25

365

2
x
5
Money withdrawn from personal savings account to
meet the cost of the new car.
6
2
9
x x=
x
=x
25
5
25
9
x was what portion of his
Now, it is not known that
25
personal savings balance.
Thus the question cannot be answered even by
combining both the statements.
Choice (4)

Shiva and the Saibaba temple is


= 17 days.
21
The number of days he goes to all the three temples is

Money realised by selling the old car =

(Numbers)
1.

365
84 = 4 days.

Hence the number of days Raju's dad goes to exactly


one temple is 121 + 91 + 52 30 13 17 4
= 204 days.
Choice (1)
4.

A + B +C + C + D + E +E +F + G = A + B + C + D + E
+F +G + C + E
(1 + 2 + 3 + 4 + 5 + 6 + 7) + C +E
Now 28 + C +E = 33.
C + E = 5
C, E could be (1, 4), (4, 1) (2, 3) OR (3, 2)there are four
possible ordered pairs of (C, E).
Choice (3)

From statement A,
xy = 18 the different possibilities are:
118
29
36

11

CASELET

Solutions for questions 8 to 12:

The trading pattern followed by each of the three traders is


as follows

Solutions for questions 1 to 3:

The arrangement of the buildings according to the given


conditions is
Yellow Blue Indigo
B
E
C

Anand
Buy
Sell
10 a.m. 3 p.m.

Bala
Chandu
Buy
Sell
Buy
Sell
10 a.m., 3 p.m. 10 a.m., 3 p.m.
11 a.m.,
11 a.m.,
12 noon,
12 noon,
1 p.m.,
2 p.m.

1.
2/3.
4.
5.
6.

Height
E
B/D
A
C
F

A
F
Orange Green

The colour of the building diagonally opposite to the


yellow coloured building is Violet.
Choice (4)

2.

The second tallest building is either B or D.


Choice (5)
The colour of the tallest building is Indigo.
Choice (2)

3.

Solutions for questions 4 to 7:

Stage
As P, Q, S and T won at least one match, R and U lost all
the three matches.
As Q, S and T lost at least one match, P won all the three
matches.
In stage-, there are a total of 9 matches and so 9 wins.
Q, S and T won two matches each.
As P (the top team in stage-) did not play against
U, P played matches against Q and R.
The ninth match was between Q and U.
So the nine matches that have taken place are as follows.
Won
P
Q
P

Lost
S
T
Q

Won
S
T
P

8.

As the direction of the price movement is not known,


the profits of Bala and Chandu depends upon the prices
at which they bought gold i.e., if they buy at lesser price
than that bought by Anand, their profits would be more,
if not, the profits of Anand would be more than that of
the other two. Hence the answer cannot be determined.
Choice (5)

9.

Anand buys the entire quantity at a single point of time,


whereas each of the other persons buy once every
hour. As the direction of movement of gold is not given,
we cannot compare the returns of Anand with the other
two persons.
Bala: Bala buys the same quantity of gold every time,
irrespective of the price.
Chandu: Chandu spends the same amount every time,
his buying depends on the price of gold at the time he
buys. The more the price, the lesser quantity he buys.
As his strategy is based on prices, whenever the prices
are changing, Chandus returns will be more than that
of Bala. But if there is no change in the price of gold the
returns of Bala and Chandu would be equal. Hence no
conclusion can be made.
Choice (5)

D
Violet

1.

Lost
R
R
R

Won
S
T
Q

Lost
U
U
U

Stage-
As each team played a total of five matches, in stage , the
matches take place between the following pairs of teams.
P T, P U, Q R, Q S, T S and R U
Given that, in stage-, three teams lost all the two matches.
Given P lost both the matches in stage-
Each of T and U won the two matches.
R and S lost the two matches.
Q also won two matches.
4.

T and U defeated P (the top team in stage-)


Choice (2)

5.

Only Q, T and U won both their matches in stage-.


Choice (4)

6.

S and U won exactly two matches in the event.


Choice (5)

7.

Q and T won exactly four matches each in the event.


Choice (5)

1 p.m.,
2 p.m.

10. On a boom day, the price of gold keeps rising, hence it


will be the least in the morning. Hence, Anand who
bought all his holdings in the morning will get the
maximum profit. Between the remaining two, Bala
bought the same quantity at every time, i.e he bought
the same quantity even at higher prices whereas
Chandu spent the same amount. Hence, Chandu
bought less quantity of gold when prices were high and
more when prices were less. Hence, Chandus returns
are more than that of Bala's. Bala will have the least
returns.
Choice (1)

Let the prices of gold at different timings be as follows.


Time 10 a.m.
Price
a

11 a.m.
b

12 noon 1 p.m. 2 p.m. 3 p.m.


c
d
e
f

We will look at the additional information given:


The quantity bought by Anand at 10 a.m. is the same as the
quantity he sold at 3 p.m. As it is given that Anand lost money,
we can ignore the quantity bought/sold and can conclude that
the price at 3 p.m. must be less than that at 10 a.m.
a>f
()
Similarly the quantity of gold bought/sold by Emma in each
instance is the same and it is given that Emma made a
profit. Hence we can conclude that (c + f) > (a + d) ()
Also using similar logic in case of David, we conclude that
(d + e + f) > (a + b + c) ()
It is given that the price increased from 2 p.m. to 3 p.m.
e<f
(V)
It is given that price at 12 noon was lower than the opening
price c < a (V)
From (i) and () we can conclude that c > d (V)
From (), () and (V) we conclude that e > b (V)
Hence a > f > e > b and a > c > d a is the highest.

12

Students who opted for Finance = a + e + d + g.


(a + e + d + g) 50% = b + f + c. (1)
Students who did not opt for HR = a + e + b.

11. The price of gold was the highest at 10 a.m.


Choice (1)
12. As d < c, choice (4) is also necessarily false.
Choice (4)

a+e+d+g=

13. The different possibilities in which they could have


booked the rooms are as follows.
Case :
102
C

103
D

104
B

101
B

102
D

103
C

104
A

f=e=

Since B booked an odd numbered room, we can


conclude that as per case , B must have booked room
number 101, in which case C would have booked room
number 103.
Choice (C)
14. It is given that two girls failed in the examination. Now
we have six possibilities in which we can select the two
girls who failed. They are as follows:
Dolly
Pass
Pass
Pass
Fail
Fail
Fail

Molly
Pass
Fail
Fail
Pass
Pass
Fail

Polly
Fail
Pass
Fail
Pass
Fail
Pass

Kelly
Fail
Fail
Pass
Fail
Pass
Pass

1
T
L
T
T

2
T
L
T
L

3
T
L
L
T

4
L
L
L
T

5
L
T
T
L

6
L
L
T
T

As exactly three of them were telling the truth, only in


case it is so. Thus Molly was the person who was
lying.
Choice (B)
15.

Finance

Marketing

a = 4k

b = 3k
e=k

It is given that books D and F was read by the same person,


A and B was not read by the same person and F and C was
not read by the same person.
The different combinations in which the books were read are
as follows:

B
C
G
H

A
D
F
E

A
D
F
G

A
D
F
H

V
B
C
E
G

A
C
G
H

B
D
F
E

V
A
C
E
H

B
C
E
H

B
D
F
G

A
C
E
G

B
D
F
H

16. As Akira read books E and G, the books that Akira read
could be either A, C, E and G or B, C, E and G. In either
case, we can conclude that Aroki did not read book C.
Choice (D)
17. As books C and E, were not read by the same person,
as in cases and V, books G and H were read by the
same person.
Choice (C)

g=3k
d = 2k f = k
c=k

Solutions for questions 18 to 25:

HR
n

It is given that n = O
g = 37.5% of (a + b + c)
i.e., g =

1
g.
3

Solutions for questions 16 and 17:

Let us denote a true statement by T and a false


statement by L (lie)
Cases
Dolly
Molly
Polly
Kelly

(2)

f = e = k.
From equation (1), a + e + d + g = 2 (b+ f + c)
a + k + d + 3k = 2 (b + c) + 2k
a + d = 2(8k a) 2k.
3a + d = 14k (3)
From equation (2), we get, 4d + 11k = 5b + a (4)
Again number of students who opted for only Finance &
HR, i.e., d was 50% of those who opted for only
Finance i.e., a.
d = 50% a.
Substituting in equation (3), we get d = 2k and a = 4k.
Substituting in equation (4) we get b = 3k.
Now (a + b + c + d + e + f + g) = 15k
15k = 270 k = 18
Exactly two = d + e + f = 4k
= 4 (18) = 72 students.
Choice (B)

Case :

Cases
1
2
3
4
5
6

Again number of students who opted for only Finance


and Marketing was 331/3% of those who opted for all
three
Number of students who opted for only Marketing and
HR = Number of students who opted for only Finance
and Marketing.

Solutions for questions 13 to 15:

101
A

5
(a + e + b).
4

3
(a + b + c)
8

Let (a + b + c) be 8k.
g = 3k
Students who did not opt for Finance = b + f + c

18. W5 and W7 are allotted a shift, one earlier than W6 and


W3 and W9 are also allotted a shift earlier than W6.
Again as W3 is allotted a shift lower than W2, if we allot
the afternoon shift for W3 and W9; W5 and W7 being one
shift earlier than W6, we will have four workers in the
Afternoon shift if W6 is allotted the Evening shift. Thus
the only shift that can be alloted to W6 is the Night shift.
The following table gives the workers and the shift they
were allotted to.

13

Morning
Afternoon
Evening
Night

Case (3)
Schumi
Sebastian
Sebastian

W2
W1, W3, W9
W5, W7
W4, W6

If Schumi beats Mclaren on all the three days, then


Mclaren will come last all the three days (not possible).
Choice (D)

Thus W8 can be allotted any shift other than the


afternoon shift.
Choice (B)
19. The following table lists down the matches and the
corresponding players who led the team as captain and
vice captain.
Match
Match 1
Match 2
Match 3
Match 4
Match 5

Captain
B
A
B
C
D

Vice Captain
A/D
C
A/D
A/B/D
A/B/C

As D refused to lead the team as captain if A or B led


the team as captain in the preceding match, we can
conclude that D can be the captain of the team only in
Match 5.
Again with D as Captain in Match 5, A must have
captained the side in Match 2 for A cannot be the
captain in Match 4.
Now with A as the captain in Match 2, C must have
been the vice captain in that match.
Thus C was the vice captain in Match 2.
Choice (C)

25. (i)

Appliances
Water Purifier
Refrigerator
AC

26. Let the runs scored by Bhajji be x

Straight drive Pull shot


Others
Total
x + 40
Pollard
x + 40
5
Dumminy (0.6) (x+ 20)
(0.15) (2x + 40) x + 20
x
= 20
x
Bhajji
4

x
= 20
4
x = 80.
Given

21. Number of matches played by Sachin is equal to that


played by Mongia. Number of matches played by
Dravid is equal to that played by Hussey. Since Sachin
has played more matches than Dravid, the average
runs must be less than 45.
Choice (A)

282
= 40.28.
7
301
= 43.
Average (Sanjay) =
7
If the average score after the exclusion lies between
40.28 and 43, then the average of Ramesh will
decrease while that of Sanjay will increase. Since,
92 is the only value lying in that range, so their score in
the invalid question is 42.
Choice (A)

22. Average (Ramesh) =

23. A wins B wins


B wins C does not win.
That implies both A and C do not win together.
That means at most one of A or C wins. That further
implies that D must win.
Choice (A)
24. Case (1)
Schumi
Schumi
Sebastian
Case (2)
Schumi
Schumi
Sebasian

Pollard
Dumminy
Bhajji

Day
Monday
Tuesday
Wednesday

For A or C:
AC must be bought before the water purifier.
Choice (B)

Mclaren
Mclaren
Schumi

Day 1 / 2 / 3
Sebastian
Sebastian
Mclaren

Sebastian
Mclaren
Schumi

Day 1 / 2 / 3
Mclaren
Sebastian
Mclaren

Both Sashi and Govind work together. This implies


Ryan and Mokambo will work together
Choice (A)

Solutions for questions 26 to 28:

20. For B:
Person

Day 1 / 2 / 3
Sebastian
Mclaren
Mclaren

Mclaren
Schumi
Schumi

Straight drive Pull shot Others Total


24
120
60
10
30
100
20
80

Maximum possible difference = 95 10 = 85


Choice (D)
27. Maximum runs scored by Bhajji through straight drive = 59.
59
2
Required percentage =
100 = 19 /3%
300
Choice (D)
28. Runs scored by Bhajji through others cannot be
determined.
Choice (D)
Solutions for questions 29 to 41:
29. (1)
(2)
(3)
(4)

All shoes are pens.


Not all pens are pencils.
All pens are chocolates.
Not all chocolates are pens.

Analyzing the options:


(A) Combining (1) and (4), we get Some chocolates
are not shoes.
(B) Combining (1) and (3), we get that some shoes
are chocolates.
Choice (D)
30.
t

Rameshs house

4
5

Total time = t +

Umeshs house

4
9
t =
t
5
5

9
t = (7 : 45 4 ) pm = 225 min.
5
t = 125 min

14

When Ramesh reaches Umeshs house, his watch was


showing 4 p.m. + 125 mins = 6:05 p.m.
Umeshs watch was showing 6:10 p.m. So, Rameshs
watch is 5 mins slower than Umeshs watch.
Choice (D)

Emmanuels rank in Physics must have been 3.


Ben got the same rank in Mathematics and Chemistry.
Remaining ranks of Ben is be 4, 4 and 12. Therefore
Ben or Emmanuel did not get the 1st rank in
Mathematics. Thus Adams rank in Mathematics was 1.
Cathy got 3rd rank in Mathematics, therefore Ben got
the 4th rank in Mathematics.
Proceeding like this we can conclude that Cathy got the
1st rank and Adam got the 2nd rank in Physics
Choice (B)

31.

T2

T1

200 x x

34.

300 x

Air Conditioners

Refrigerators
y
b

24
x 300 x + y
=
y 200 x + y

Let

x
=2
y

32.

Swimming Running Cycling Walking Total


5
6
15
1
6
5
2
4
14
1
18

In case of Z:
A sum total of 18 is possible when two 6s and one 5
is there in addition to 1.

It is given that at least 40 families own both a


Refrigerator as well as a Air conditioner.
b + d is at least 40.
We have to find the maximum value of a.
a will be maximum when (b + d) is minimum.
i.e., when (b + d) is 40.
Now a+ c+ (b + d) + 24 + 20 + 26 = 120.
a + c = 120 70 (b + d)
a + c = 10.
The maximum value of a is 10, when c is 0.
Thus at most 10 families owes a refrigerator and a LCD
TV but not an Air Conditioner
Choice (A)
35. The lectures and the days on which they deliver the
lectures are tabulated in the following figure.

Lecturers
Swimming Running Cycling Walking Total
2
5
2
6
15
1
6
5
2
4
3
14
6
6
1
5
18

The above table gives the ranks obtained by the four


persons in the four events.
In case of X, a sum total of 6 is possible only if
1 + (1 + 3 + 1) is there. He has to get rank 3 in cycling
and rank 1 in each of running and walking.
Choice (A)
33.

Name
Adam Ben Cathy Dimitry Emmanuel
Subjects
Mathematics
Physics
Chemistry
Biology
Total

LCD TVs

then y > 100 and x > 200. This is not possible so only
one value exists
Choice (A)

W
X
Y
Z

20

26

400 2y = 300 y
y = 100
x
If
>2
y

W
X
Y
Z

1
2
3/5
5/3
11

4
5
4
2
15

3
1
5/3
3/5
12

2
4
1
1
8

5
3
2
4
14

Since the sum of the ranks of Dimitry was 8 and he got


the same rank in Chemistry and Biology, his ranks in
Chemistry and Biology was 1.
Therefore Dimitrys rank in Mathematics was 2 which
was the same as Emmanuels rank in Chemistry.

Monday
L1

Tuesday Wednesday Thursday


L4
L2
L1
L3

L4 can deliver the lecture either on Tuesday or on


Wednesday.
Now, if L4 delivers his lecture on Wednesday, then L3
cannot deliver his lecture on any of the given days.
[Since L3 delivers a lecture only if L2 delivered a lecture
on the preceding day and L3 and L4 do not deliver
lectures on consecutive days.]
Thus L4 delivers the lecture on Tuesday.
Now, the only day on which L3 could have delivered the
lecture was Thursday
Choice (D)
36. The different ways in which the committee can be
formed is as follows:
1. B2 B4 B5 G2 G3
2. B1 B4 B5 G1 G3
3. B1 B4 B5 G2 G3
4. B1 B3 B5 G1 G3
5. B1 B3 B5 G2 G3
6. B2 B3 B5 G2 G3
Therefore there are six ways in which the committee
can be formed
Choice (D)
37. In order to have the total machining time as minimum,
none of the machines must be idle at anytime and the
total time taken must be 10 hours. (i.e., higher of the
total machining times in the two machines).
Let us consider the answer options and check if it is
possible.
Option A

15

M1

M2

P3

P1

Case 1 M9 belonged to Germany.

Duration
2
3

P1

P2

P2 (4)

P3 (5)

UK Germany France Switzerland Turkey


M6
M9
M2
M1
M7.
M8
M10
M4
M3
M5
Case 2 M9 belonged to Turkey.
UK
M6
M8

Total = 10
Option B

Germany France Switzerland Turkey


M5
M2
M1
M7
M10
M4
M3
M9

(As M5 did not belong to France or Switzerland)


Thus M10 belonged to Germany
Choice (C)

M1

M2

P1

P2

P3

P1

P2 (4)

P3 (5)

Duration

Total = 10
In case of option (C), if product P2 is machined in M1
before Product P1, since P2 takes 4 hours in M1, it can
be done as follows:
Duration 0 4
47
79
M1
P2
P1
P3
Duration 0 5
57
7 10
M2
P3
P2
P1
The total time taken is again 10 hours
Choice (D)
38.

Football

Cricket
a
d

41. As per the conditions given the different ways in which


the terms for the two contest can be selected as
follows:
Debate: PVQ PVT PVS PVR PVT PVRPVR PVS PVU PVU PVR
Elocution:PURPUR PURPUT PUS PUSPUT PUT PQS PQT PUQ

e
g

40. It is given that sum of the costs of the gifts bought by


Sneha and Sushma was equal to the cost of the gift
bought by Shikha.
We have four possibilities which satisfies this.
Sneha Sushma Shikha Sushmita
Case 1.
800
1200
2000
2800
Case 2.
1200
800
2000
2800
Case 3.
800
2000
2800
1200
Case 4.
2000
800
2800
1200
Again the difference between the cost of Sushmas gift
and Sushmitas gift was equal to the cost of Snehas
gift. This is satisfied only in Case 3. Thus the cost of the
gift bought was Shikha was `2800 and she bought a
pair of shoes.
Choice (D)

Debate: PVS PVT


Elocution:PUQ PUQ
Option A is false as can be seen in the following cases:
Debate: PVR PVS PVT
Elocution:PUQ PUQPVQ
Option B is true. If V and U are in the same category it
must be for Debate. We know that U being in debate
implies R is not in elocution. Again since only one
among S and T can be selected for a particular
category, Q must be selected.
Option C is true as can be seen in the following cases:
Debate: PVR PVS PVT
Elocution:PUQ PUQPVQ
Thus only statement given in option A is false
Choice (A)

e
Hockey

Those playing exactly 3 games = g


Those playing exactly 1 game = a + b + c
Those playing exactly 2 games = d + e + f
Those playing at least 2 games = d + e + f + g
It is given that d + e + f + g = 18 (1) and
(a + b + c) + (d + e + f + g) = 30 (2).
Therefore a + b + c = 12.
Now (a + b + c) = 3 (g).
or, 3g = 12
or, g = 4.
Therefore d + e + f = 18 4 = 14.
Thus the number of members playing exactly two
games is 14
Choice (B)

NETWORKS
1.

The cost incurred will be minimum when the distance


travelled is the minimum.
The distance travelled is minimum when he takes a bus
going via A E D F G H.
The minimum cost incurred by him = 5 + 8 + 4 (10)
= `53
Choice (3)

2.

If the road connecting A to E is under repair, then to


incur minimum cost, one must board a bus going via the
route A D F G H. Since the total distance travelled
along this route is the least.
The cost incurred = 5 + 8 + 4 (12) = `61
Choice (3)

39. We can list down the names of the countries and the
athletes belonging to them as follows.
Countries UK Germany France SwitzerlandTurkey
1.
M6
M7
M8
2.

It is given that M6 and M8 belonged to UK where as M7


belonged to Turkey.
Now M5 and M9 were from different countries and M9
did not belong to France or Switzerland.
So M9 belonged to either Germany or Turkey.
Now M1 and M3 belonged to the same country and so
did M2 and M4.
Let us consider two cases.

QBR (Miscellaneous)
1.

Given that the number of people rightly reported is 275.


This includes people under C3 and C4.
C3+ C4 = 275 .......... (1)

16

Number of people wrongly reported = 450 275 = 175


C1+ C2 = 175 . (2)
Given the number of infected people is 50% that of
non-infected
Number of infected people = 150 = C1 + C4 . (3)
And the number of non-infected people = 300
= C1 + C3 . (4)
Required difference is between C2 and C4, obtained
by (2) (3) C2 C4 = 175 150 = 25 Choice (2)

Given that for every `2 increase in the selling price


per ball, the number of balls sold decreases by 20.
If the selling price of each ball is increased k times,
selling price = `59 + 2k.
Profit per ball = (59 + 2k) 50 = 9 + 2k.
Number of balls sold = 700 20k.
Profit obtained = (9 + 2k) (700 +20k).
Profit = 10 ((9 + 2k) (70 2k))
= 10 (630 18k + 140k 4k2)
= 10 (630 (4k2 122k))
2
2

61

61
= 10 630 2k

2
2

61
61

= 10 630 + 2k

2
2

61
61
Profit is maximum when 2k
= 0 2k =
2
2
But since k is an integer, 2k must be an integer.
60
62
2k can be taken to be
or
2
2
If 2k = 30, profit = 10 ((9 + 30) (70 30)) = 10 39 40
= `115600
If 2k = 31, profit = 10 ((9 + 31) (70 31)) = 10 40 39
= `115600
For maximum profit,
Selling price = 59 + 2k = 59 + 30 = `89 or 59 + 31 = `90
When selling price is `89, balls sold = 700 300 = 400
When selling price is `90, balls sold = 700 320 = 380
Of the given choices, only (A) satisfies. Choice (1)

2.

3.

4.

From the choices only option (a) indicates low per


capita income and low happiness quotient.
Choice (1)

5.

From the choices only option (b) indicates high


happiness quotient and high per capita income.
Choice (2)

Solution for question 6:

DI (Miscellaneous)
1.

Solutions for questions 4 and 5:

Total yield from scheme = 0.25 (3) + 0.55(80) +


0.2(100) = 7.5 + 44 +20 = 56.5
Therefore the total yield from scheme II was also 56.5.
Let the probability of the bearish market be p.
The probability of the bullish market = 1 0.4 P
= 0.6 P
Now, p ( 10) + 0.4 (60) + (0.6 p) 100 = 5605
110 p = 60 + 24 56.5
p = 0.25
Choice (3)
Scheme 1
Market
conditions

Probability

Yield percentage

Bearish

0.2

30

Steady

0.45

80

Bullish

0.35

100

The yield from scheme = 0.2 ( 30) + 0.45 (80) + 0.35


(100) = 6 + 36 + 35 = 65
Increase in the total yield from scheme 1
65 56.5
Choice (4)
=
100 = 15%
56.5

6.

Written

WE

Interview

Essay
writing

GD

Rahul

Ramya

The cumulative score of Rahul is


5 0.3 + 3 0.1 + 2 0.25 + 3 0.1 + 4 0.25= 3.6
The cumulative score of Ramya is
5 0.3 + 1 0.1 + 3 0.25 + 4 0.1 + 3 0.25 = 3.5
The required difference is 0.1
Choice (1)
Solutions for 7 and 8:
7.

8.

The expected pay-out for Raju is


80 0.5 + 40 0.3 20 0.2 = 48

Choice (2)

The expected pay out for Ramu is


80 0.5 + 60 0.3 20 0.2 = 54
After the change of probability the expected pay out for
Ramu is 80 0.3 + 60 0.5 20 0.2 = 50
4
The required percentage decrease is
100
54
= 7.4%
Choice (1)

Solutions for question 9:


9.

The hotel cost for Ramu = $600


The cost incurred for city tour = $40
The cost incurred for tour of the Hunters valley = $35
Hence the total cost incurred by Ramu = $675
Choice (3)

10. Given that there has to be a male in every group.


Hence only three groups can be formed.
Also given P, S are in same group and each group has
atleast one JSE and one SSE.
As both P and S are JSE. The team should have one SSE.
Given R is in a group of 3 people
Hence the three groups should have 3, 3 and
2 members in each.
Now considering the condition one JSE and one SSE in
each group we get the following possibilities.

Group 1:
Group 2:
Group 3:

(i)
R XY
ZPS
WQ

(ii)
RXY
ZQ
WPS

From the above possibilities we can conclude that X


should definitely be a member of a group which has 3
people.
Choice (1)

17

Solutions for questions 11 and 12:


11. Let the value of the number in column 'b' and row 'd' be
'x' and that in column 'b' and row 'b' be 'A'.
1
(A + 26 + x)
Given x =
3
1
2x = A + 26 x = ( A + 26)
2
As we know the grid contains only integers
Therefore x should be an integer.
Hence A should be even.
From the choices only 16 is possible Choice (4)
12. Given the numbers in column 'a' are squares of the
prime numbers, starting with the first odd prime number.
Hence the numbers should be 32, 52, 72. 112 and 132.
Thus, the required sum is 32 + 52 + 72 +112 + 132
= 373
Choice (4)

17. By observing we can easily find that R and S have the


same visibility index.
Choice (3)
Solutions for question 18:
18. The points of A = 1 30 + 2 20 + 2 10 + 1 5 = 95
The points of B = 2 30 + 1 20 + 1 10 + 3 5
= 105
The points of C = 3 30 + 2 20 + 1 10 + 1 5
= 115
The points of E = 1 30 + 2 20 + 1 10 = 80
The winner is C
Choice (3)
Quant SI CI
1.

Solutions for questions 13 and 14:

Given total number of sarees = 400


The ratio of Kanchipattu, Benarasi and Mangalgiri sarees is
5:3:2
The number of Kanchipattu sarees = 200
The number of Benarasi sarees = 120
The number of Mangalgiri sarees = 80
Given on day 1 he sells 20% of the total which is 80 on day
2 he sells 200 and on day3 he sells 120.
Also on each day he sells the sarees in the same ratio as he
bought i.e 5 : 3 : 2
K
40
100
60
200

Day1
Day2
Day3
Total

B
24
60
36
120

M
16
40
24
80

Total
80
200
120
400

13. On the 3 day he sold Benarasi saree at `480.


Total amount received by him on the 3rd day is
60 350 + 36 480 +24 375 = `47280
Choice (2)
rd

2n

10

10000.
Interest from scheme 3, 3= 10000 1 +
200

For n = 8, 3 = `11829 & 2 = `12,000


For n = 9, 3 = `14066 & 2 = `13,500
Choice (2)
After 9 years, 3 > 2.
2.

Solutions for question 15:


15. The population of China in 2009 = 1.6 billiion
As it increases by 12% per annum it becomes
5.56 billion in 2020
The population of China in 2020 is 15% of the total
population
5.56
Hence the total population is
= 37 billion
0.15
Choice (2)
Solutions for questions 16 and 17:
16. The graph gives visibility index of 26 people.
The visibility index of 14 people are more than U.
14
Hence the required ratio is
100 = 53.8 54%
26
Choice (2)

Scheme 1 Scheme 2
80
150
166.4
300
259.7
450
360
600
469
750
587
900
714
1050
851
1200

Scheme 3 Scheme 4
102.5
80
216
166.4
340
259.7
477
360
629
469
796
587
980
714
1183
851

In scheme 4, amount at the end of the year = 1000 (1.2)


= 1200.
Amt. remaining after paying the administrative charges
= 0.9 (1200) = 1080
Amt. at the end of the second year = 1080(1.2) = 1296
Amt. remaining after paying the administrative charges
= 1296 (0.9) = 1166.4
This scheme is similar to scheme 1.
Therefore scheme 2 produces the maximum interest at
the end of 8 years.
Choice (2)

1
(400) = 100
4

The ratio of the damaged sarees of each type is 5 : 3 : 2.


Hence the number of damaged Kanchipattu sarees
= 50
The number of the damaged Benarasi sarees = 30
The number of the damaged Mangalgiri sarees = 20
He sold all the damaged sarees at 20% loss.
Total amount = 50 280 + 30 320 + 20 300
= `29600
Choice (3)

Let us consider the interests received by him from the


four schemes across the year with `1000 invested in
each scheme.

Year 1
Year 2
Year 3
Year 4
Year 5
Year 6
Year 7
Year 8

14. 25% of the total number of sarees were slightly


damaged.

Let the number of years after which his interest in


scheme 3 will be more than his interest from scheme
2 be n.
15
Interest from scheme 2, 2 =
(10000 ) n = 1500n.
100

LA (Venn Diagram)
1.

Let the number of students who applied for all three


examinations be x.
The number of students who applied for at least 2 of the
3 examinations = 36 2x
FMS

CAT
9x
x
15x

12x
XAT

It is given that 25% (36 2x) = x


36 2x = 4x
x=6
So the completed venn diagram will be as follows.

18

We have the following information.


The Hindu was read by 64 families.
A + d + e = 55 (1)
The times of India was read by 48 families.
B + d + f = (2)
The Telegraph was read by 45 families.
C + e + f = 36 (3)
Adding equations (1), (2) and (3), we get A + B + C + 2
(d + e+ f) = 130
Again A + B + C + d + e + f = 90
d + e + f = 40
A + B + C = 90 40 = 50
Therefore exactly one newspaper was read by 50 families.
Choice (3)

FMS

CAT
3

18
9

11
XAT

Number of students in the class = 18 + 3 + 6 + 9 + 6 + 11


= 62
Choice (3)
Solutions for questions 2 and 3:

Speak in
English (500)

Speak in Hindi (480)


e=
240

d = 120 t =
60
40
C = 180

Own a car (400)


As per the data provided in the question,
A + e + d = 360 (1)
B + e + f = 380 (2)
C + d + f = 280 (3)
d + c = 24 (4)
(3) (4) gives f = 40
Again d + 120 = 180
d = 60
c = 180
From (1), A + e = 300 and from (2), e +B = 340.
Now A + e + e + B = 400 [ the total number of persons
= 800]
A + e + e + B = A + e + B + e = 300 + 340.
e = 240
A = 60 and B = 100
2.

The number of persons who can speak in both Hindi


and English = 240 + 120 = 360
360
The required percentage =
100 = 72%
500
Choice (3)

3.

The proportion of people in the locality who do not own


B+e+ A +d+C
a car or cannot speak in English =
800
640
= 0.8
Choice (4)
=
800

4.

A + B + C + D +E +F = 120 (21 + 9) = 90
The Times
of India

The Hindu

A
e

9
f
C

The Telegraph

21

Solutions for questions 5 and 6:

Given GT = 300 1
GT 300
a + d + f + g = 60 2
H
S
b + d + e + g =120 3
b
c + e + f + g = 180 4
d
a
a+d=f+g5
g
e
f
a=f=06
c + f = 30 7
c
Equations 5 and 6 d = g
n
C
Equations 6 and 7 c =30
Using GT formula a + b + c + d + e + f + g + n = 300
0 + (b + d + e + g) + 30 + 0 + n = 300
n = 60
Using equations 2, 5 & 6, we get d = g = 30.
5.

g
30
100 =
100 = 10 %
GT
300
Choice (2)

From 4 and c = 30, we get e = 120


Substituting in 2, Eqn. we get b = 30
Therefore b + n = 90.

6.

Choice (4)
7.

Given, of the 300 students, 70 choose MS


230
Hence 230 choose MBA.
Given g = 20 1
M
F
n=0
b
d
a + d + f + g = 100 2
a
g
b + e + d + g = 150 3
e
f
d = 2f = g
d = 20, f = 10 4
c
From 1, 3 and 4, we get
n=0
H
b + e = 110
GT = a + b + c + d + e + f + g + n
230 = 100 + b + e + c
c = 20
Choice (3)

8.

Given the number of children who buy T & J (A) = 16


The number of children who buy C & H (B) = 26
The number of children who buy B & B (C) = 34
We know A + B + C = Ex1 + 2Ex2 + 3Ex3
Where Ex1, Ex2 and Ex3 denote the number of children
buying exactly one, exactly two and exactly three toys
respectively
Given every child buys exactly 2 toys
Hence Ex1 = Ex3 = 0
76 = 2Ex2
Ex2 = 38
Hence there are 38 children who visited the shop
Choice (4)

Solutions for questions 9 and 10:

Given P = 24 (1)
Q = 36 (2)

19

R = 29 .. (3)
P
a e b Q
S = 25 .. (4)
e + l + o + n = 12 .. (5)
f + l + k + o = 18 (6)
R
f
l g
g + m + l + o = 16 .......... (7)
c
K = 8 and l + o = 6 .. (8)
k o m
h
From (5) & (8) we get
i
j
n
d S
e+n=6
P
From (1) & (6) we get
a+e+f+l+k+o+j+n=
24
a+e+n+j=6
but e + n = 6 a = j = 0
As the people who like S also like R.
Hence d = i = n = j = 0. h + m + k + o = 25
Therefore e = 6 and b = 4.[As b + (g + l + m + o)= 36, where
(g + l + m + o) = 16]
From (6) & (8) f = 4
From (7) & (8) g + m = 10
From (3) c + h + g +m + l + o + f + k = 29
c + g + l + f + 25 = 29
c=g=l=0
Therefore h = 1, m = 10 and o =6
9.

Therefore the difference between the highest marks


obtained is 20.
Choice (1)
4.

Correct
Wrong
Score

Stores

Ranking as
per price.

A
B
C
D
E
F

3
6
1
2
5
3

5.

6.

Solutions for Questions 2 to 4:

The least score obtained by a person in the four papers


can be obtained as follows.

Correct
Wrong
Score

Paper 1 Paper 2
4
3
0
1
60
45

Paper 3
1
3
20

Paper 4
4
0
80

Therefore the minimum net score that the person can


get is 165.
Choice (4)

Paper 1
2
2
20

Paper 2
4
0
110

After the given swapping the final arrangement will be


as follows:
1

After the given swappings the final arrangement is as


follows:
1

7.

The proportion of residents who prefer watching movies


PA is 0.65
The proportion of residents who prefer surfing the net,
PB is 0.68
The proportion of residents who prefer doing both, PAB
is 0.61
The proportion of residents who prefer at least one
between watching movies or surfing net is PA + PB
PAB
= 0.65 + 0.68 0.61 = 0.72
Proportion of residents who neither watch movies nor
surf net is 1 0.72 = 0.28

8.

Let the population of A be 3k.


Population of B is 5k, that of C is 3k and that of D is
4k.

Paper 3
4
0
100

Paper 4
1
2
30
Total score = 140

Solutions for questions 7 to 9:

Total score = 120


Correct
Wrong
Score

From the choices only "W is in the 9th seat" is correct.


Choice (2)

3.

Correct
Wrong
Score

Choice (2)

Therefore X will be seated beside U

Therefore pizza store D got the third lowest net score.


Choice (2)

2.

Paper 4
4
0
80

The final arrangement of the persons in the 9 seater van


was as follows:

Ranking of the stores and the total net scores for the
pizza stores are as follows.
Ranking as
per delivery
Total net score
time
3
3(0.7) +3(0.3) = 3.0
2
6 (0.7) +(0.3) = 4.8
1
1(0.7) + 1 (0.3) = 1.0
6
2(0.7) + 6(0.3) = 3.2
3
5(0.7) + 3 (0.3) = 4.4
5
3(0.7 + 5(0.3) = 3.6

Paper 2
2
2
10

Solutions for questions 5 and 6:

LA (Miscellaneous)
1.

Paper 1
4
0
60

Total score = 150

The number of people who like only Q = 14


Choice (4)

10. The number of people who like all 4 movies is 6


Choice (2)

The least score is obtained when the person attempts


the following 3 papers.

20

No. of residents who prefer chatting with friends


in A is 0.36 3k = 1.08k
in B is 0.45 5k = 2.25k
in C is 0.32 3k = 0.96k
in D is 0.25 4k = 1.00k
Highest number is in B
Choice (2)
9.

No. of residents who prefer chatting with friends was


calculated in the previous question.
The average number of residents who prefer surfing net
1.9k + 2.75k + 2.04k + 3k
= 2.4275k
=
4
2 colonies have more than the average number.
Choice (3)

10. The different ways in which oil can be transferred from


tank B to tank H are
1. B E A F D C G H
2. B E A F G H
3. B E C A F G H
4. B E C G H
5. B E D A F G H
6. B E D C A F G H
7. B E D C G HS
Thus there are seven possibilities in all.
Choices (c)

Hence the probability of Rajini taking something home


4

is

C1 2
= [since all the baskets are equally likely to
C1 3

get selected by Rajini]


Solutions for question 14:

14. Let us consider P = 1. When there is one goat and one


tiger then the tiger eats the goat and gets transformed
into a goat and stays happily in the forest.
Now if P = 2 when there are two tigers. Now, if one of
the tigers eat the goat then it gets transformed into a
goat and then the second tiger would kill it. Hence when
two tigers are there they would not kill the goat.
Let P = 3. When three tigers are there. One of the three
tigers kills the goat and becomes a goat. Hence the
remaining tigers would not kill the goat.
Hence when the tigers are odd numbered then they
would kill the goat, else the goat is not eaten by any
tiger.
Choice (2)
Solutions for questions 15 and 16:

Given a + g = 12
b+g=8

GT
T

11. Based on the conditions given in the question, we get


the following possibilities.

Arun
Varun
Kiranmala
1. India-Day-to-Day India-Every-day India-These-days
2. India-These-Days India-Day-to-Day India-Every-day
We can conclude that Varun did not subscribe to
India-These Days.
Choice (3)
12. If Kiranmala did not subscribe for India-These Days,
then Varun subscribed for India-Day to Day.
Choice (1)
Solutions for question 13:
13. Given out of the six baskets four baskets have either
gold or silver.

Choice (2)

P
g

b
n

15. g = 4
The number of days that Ram learnt an instrument is
a + b + g = 16
Choice (4)
16. Given a = 6
Hence g = 6 and b = 2
He learnt an instrument on a + b + g i.e 14 days in all.
Choice (3)
17. From the given pattern we can understand that it is a
cyclic pattern.
Hence the input is repeated in every 7th step.
Therefore step 28 would be the same as the input
Choice (4)

LA (Circular Arrangements)
1.

The sitting arrangement was as follows

B
F/H

A/H

H/A

H/F

Therefore from the above possibilities we can conclude that H is sitting opposite to C or D.
2.

From option (a)


T

W
Q

U
V

Y
X

Choice (4)

U cannot see P, W and Q, so option (a) is not correct


From option (b)
P
T
W
Q

R
Y
X

21

All the conditions are satisfied, so (b) can be the answer

Hence we can say that the Mumbai co-ordinator is opposite


the Delhi co-ordinator.
Choice (1)

From option (c)


P

Solutions for question 4:

Given R and T sit together. We can arrange them in 2


ways.
P and S do not sit together, so we can arrange them in
6 ways.

4.

V
U

X
Y
T cannot see y, so option (c) is not correct
From option (d)
P
T
Q
W

T/R
R/T

S
S

V
Y

T/R

Bring the line below the diagram U cannot see P, so


Choice (4) is not correct
Choice (2)

T/R
R/T

R/T
P

Solutions for questions 3:

Using the first clue we can draw the figure as follows:

H
T/R
R/T
We can arrange the remaining two employees Q & U
in 2 ways.
Therefore, the total number of ways = 2 6 2
= 24 ways

M
B

Alternate solution:

Considering R and T as a single unit, we get 3 units


(R, T), Q and U which can be arranged around the
table (3 1)! 2! ways [2! since RT can be arranged
among the themselves] Now the remaining 2 persons
can be placed in 2 of the 3 positions in 3C2 2! ways.
Therefore the total number of arrangements = (3 1)!
Choice (B)
(2!) (3C2) 2! = 24 ways.

Using the 2nd clue we get,

M
B
LA (Distribution)
Solutions for questions 1 to 3:

From the given data we can conclude that the twelve persons were living in the building as follows.

Guitarist Singer Instrumentalist Drummer Singer Drummer


F

Floor 1

Floor 2

Floor 3

E/B

G/E

Floor 4 Floor 5 Floor 6

Key board
Singer Guitarist Guitarist Singer Instrumentalist
player
Q
Floor 7

Floor 8 Floor 9 Floor 10 Floor11

A
Floor 12

22

1.

The four singers were S, T, C and P. Therefore only


one male singer was there in the band. Choice (1)

2.

Above the floor in which G lived there were 6 floors or 8


floors.
Choice (4)

3.

S lived in the second floor.

on November 15th, 1998.and the one who got married


on January 3rd, 1999. And it was 49 days. Choice (1)
6.

Based on the conditions given, the groups are as


follows:
Jalan, Kokila and Kadambar
Jagan, Kavya and Kavita, Jeevan, Kavya and Kekul
Therefore Jagan is in the same group as Kavya and
Kavita
Choice (1)

7.

Given B went to college on Thursday and did not teach


Physics. As A, B and C did not teach Physics, we can
conclude that D taught Physics.
As A and C went to college on consecutive days it can
be either on Monday and Tuesday, or Tuesday and
Wednesday.
They cannot go to college on Tuesday and Wednesday
as Physics is taught after Chemistry.
Hence A and C go to college on Monday and Tuesday
respectively.
Biology is taught on Monday.
Choice (1)

Choice (2)

Solutions for questions 4 and 5:


4.

From the given data, the only possibility is


1993 November 5th
1994
1995
1996 February 29th
1997 April 23rd, October 15th
1998 November 15th
1999 January 3rd.
Both the couples who got married in the same month
got married in the month of November. Choice (3)

5.

The least difference between the marriage dates of any


two couples was between the couple who got married

Solutions for questions 8 and 9:

Given Team I scored maximum number of points, 364 is at the 15th place and Team A got 361 points.
Given the sum of the points scored by teams at (13 + 14 + 15) is 1046.
364 + 361 + x = 1046
x = 321
Hence Team A is in the 14th place.
As team C got 218 points and is in the 10th place and Team O got 251 points.
Hence Team O should be placed between 11 -13.
Given the ascending order of teams according to their points is O J F.
Hence Team O is in the 11th place, team J is in the 12th place and Team F is in the 13th place with 321 points.
Now we know the points of the teams in the 10th and the 11th place. Hence the points of the team in the 12th place is 284 points
(from 10 + 11 + 12 = 753).
As Team N and team B got 108 and 165 points respectively less than team F. Team N got 213 points and Team B got 156 points.
The clue 7 + 8 + 9 = 590 implies that we can calculate the points of the third team in the above group.
[We know N is one of the teams in the group]
Hence 213 + 182 + x = 590.
x = 195 points.
Hence Team N is in the 9th place.
The clue Team E got 18 points less than Team N, implies that team E is in the 8th place.
From the clue 4 + 5 + 6 = 412, we know B is one of teams in the group.
Hence 116 + 156 + x = 412
x = 140
Hence Team B is in the 6th place.
The clue Team H got 4 points more than Team M, implies that
Team K is in the 5th place.
From the clue 1 + 2 + 3 = 302, we get
96 + x + 4 + x = 302
x = 101
Hence Team M got 101 points and is in the 2nd place.
Therefore Team H got 105 points and is in the 3rd place.
The final arrangement is as follows.
Team
Points

1st
L
96

2nd
M
101

3rd
H
105

4th
D
116

5th
K
140

6th
B
156

8.

The required difference is 284 140 = 144


Choice (2)

9.

The position of team E is 8th .

7th
G
182

Choice (2)

Solutions for questions 10 and 11:

Given P wears the Orange shirt and he orders Sprite and


the person wearing the Green shirt orders Pepsi.
From the clues, we know R wears the Red shirt, U wears the
Blue shirt, T drinks Thumsup and Q drinks Coke.
Hence R, Q, T and U neither wear Green nor order Pepsi.
So the Green shirt is worn either by S or V. But from the
clue, the person wearing Green shirt is the person who

8th
E
195

9th
N
213

10th
C
2182

11th
O
251

12th
J
284

13th
F
321

14th
A
361

15th
I
364

orders Fanta and V orders the same dish. Hence we know


that V does not wear the Green shirt. Therefore S wear the
Green shirt and he orders Pepsi.
Let us tabulate the data.
P
Q
R
S
T
U
V

Shirt
Orange
Red
Green

Drink
Sprit
Coke
Pepsi
Thums up

Blue

23

It is given that the total work required to complete work


3 5
7 36
in A2 =
N D + N D =
(ND)
4 6
8 24
This will be true for infinite values of N.
Thus a unique value of N cannot be determined from
the given information.
Choice (4)

Now from the last clue the person wearing the Violet shirt
ordered Maaza.
Hence V ordered Maaza and is wearing the Violet shirt.
10. S drinks Pepsi.

Choice (2)

11. V Violet shirt, Maaza is the correct combination.


Choice (1)

Line + Bar graph


Solutions for questions 12 and 13:
Solutions for questions 1 to 4:

From the first and the last clue we get the following
arrangement.

O
SA

1.

L
SL

From the third clue we know that the cricketer from India is
at the extreme left end of the row and he plays for RCB.
Also P is to the immediate right of him who plays for DC.

P
IND
RCB DC

O
SA

2.

L
SL

nd

Now from the 2 clue we know that N is to the immediate


left to the player who plays for KKR.
From the above consideration we get the final arrangement
as

M
IND,
RCB

P
N
AUS, ENG,
DC
MI

O
SA
KKR

L
SL
DD

Choice (1)
Choice (4)

15. From the choices we can say that X, U and Q can be


the first person to give the presentation in 2nd group but
V cannot be the person.
Choice (4)
16. Given R attends the Physics tuition, Q attends the
Maths tuition. Also P and U same tuition, T and V
attend same tuition and S does not attend the same
tuition as Q.
Hence S should attend a tuition with at least
2 students in it. i.e. S should either attends Physics or
Chemistry as there should be at least 2 students in
each tuition. Q should attend a tuition to which
3 people go.
Choice (4)
Quant ERPV

Persons
killed

Persons
injured

Trucks

19500

6630

12870

0.52

Bus

13500

4050

9450

0.43

Car

12000

4200

7800

0.54

Two
wheelers

21000

8400

12600

0.67

Others

9000

4050

4950

0.82

3.

The number of people who got injured by car accidents


was 7800
Choice (2)

4.

The number of persons killed in truck is 6630 and


number of persons injured in other type of accidents is
4950.
The required difference is 1680
Choice (4)

Solution for questions 14 and 15:


14. Given X gave the presentation before T, R gave the
presentation before V but after U, also W gave the
presentation after P and S but before U and X.
So, Q, U, X, T, R and V (need not be in the same order)
conducted the seminars after W. Hence W should be
giving his presentation in group 1 with P and S.
Choice (1)

Total
accidents

x = the ratio of the persons killed to the persons injured.


The required ratio is the highest for other types of
accidents.
Choice (2)

12. M plays for RCB and comes from India.

13. The player from Australia plays for DC.

The number of accidents caused because of two


28
75000 = 21000.
wheelers is
100
The accident severity index for two wheelers is 40.
i.e. for every 100 accidents, 40 persons are killed.
So for 21000 accidents 210 40 = 8400 persons are
killed.
Choice (3)

DI (Distribution)
Solutions for questions 1 and 2:

As policies mature in between 1997 and 2002 and a policy


matures on Feb 29th, the policy should mature on 29th Feb
2000.
Now the policy which matures on Jan 10th matured after
Feb 29th. Hence the policy on Jan 10th can be in 1997 or in
2002.
But from one of the clues the policy on May 21st is the last
and the Sep 17th policy is immediately before the May 21st
policy.
Hence the policies on Sep 17th and Jan 10th should mature
the same year.
The policy on August 8th is before Feb 29th.
Hence August 8th policy is for either 98 or 99.
The final arrangement is as follows.
1st

1.

The ratio of the floor areas of A1 and A2 is 1 : 4


Now the total work required to complete the work in
1
9
N 5
A1 = N (D) +
ND, where D = no. of
D =
6
4 6
24
hours
The total work required to complete the work in A
9

ND .
must be 4
24

2nd

3rd
4th
5th
6th
th
29
Feb
24th Aug' 8th Aug
10th Jan 17th Sep May 21st
2000
1997 1998/1999
2001
2001
2002
1.

The third matured policy is on 29th Feb 2000


Choice (1)

2.

In the year 2001, Atul receives money from two policies.


Choice (2)

24

LA(Linear Arrangement)
Solutions for questions 1 to 3:

Given S is in 3rd place from the left end and the positions of F, Q, G are also given.

S
UK

As two of F, Q, G are from USA and there is atleast one person between any two friends from USA, we can say that F and G
are from USA.
Also the extreme ends are occupied by friends from USA.

F
S
UK USA

USA

G
USA

USA

USA

Now the friends from UK are separated by atleast four friends. So the other friend from UK can come either in the 2nd or the
4th position from the right. But from one of the clues P is from Australia and is in between and J. Hence we can get it as.

USA

Aus

F
S
Q
UK USA Aus

G
USA UK

P
/J
J/
USA Aus USA

Now as R is adjacent to and the friends at extreme ends are of different gender, we can get the final arrangement as follows.

T
H
USA Aus

S
F
Q
UK USA Aus

G
R
USA UK

J
USA Aus USA

1.

F, G, and J stay in the USA. Therefore four female


friends stay in the USA.
Choice (4)

2.

Three friends are in between the friends from the UK


Choice (1)

3.

T is the only male friend from USA.

Choice (2)

Solution for question 4 and 5:

Given from (i) and (iv), the arrangement would be

From the (iii) clue, U and V have 2 persons in between


them.
Hence we have only one possibility.[since there is only one
person between Tarun and Qureshi]

U/V

V/U

From the (ii) clue we get the final arrangement as

S
4.

5.

U/V

V/U

Now D got the 4th rank D gets less than C. But the
condition that one pair should have same marks is not
satisfied. Hence E should get the highest marks.
So E > C > A
Now D cannot have more marks than C as D should have
marks less than 3 students, hence C > D.
As the first and the last ranked students do not have same
marks as any other student, B should get the same marks
as 'C'.
Hence all the conditions are satisfied and the final
arrangement is E > C = B> D > A.
1.

E got the highest marks.

Choice (4)

2.

B and C got the same marks.

Choice (2)

3.

The descending order is ECBDA.

Choice (1)

Solutions for questions 4 and 5:

Given at least 2 movies were released before R and there is


one movie released between S and T.
Hence R cannot be released on the 3rd or the 4th week of the
month. It can only be released on the last week.
Therefore Q is released on the 4th week. As S and T have
one movie released between them, P is released on the 2nd
week.
Hence the final order would be as S/T, P, T/S, Q, R.

Raju is 3 places away to the left of Qureshi.


Choice (4)

4.

R is released last.

Pradip is to the immediate right of Shyam.

5.

Only one movie is released before P.

Choice (3)
Choice (1)

Choice (3)
Line Graph + Table
LA (Sequencing)
Solutions for question 1:
Solutions for 1 to 3:
1.

From the clue, D got less than 3 other students hence D got
the 4th rank.
From the other clue E > C > A.
Let us assume B got the highest marks. Then the order is B
> E > C > A.

From the table we can identify that the profit/ton in 2000


is the highest for Steel. From the line graph it is clear
that the production of Steel is the highest.
Hence the profit of Steel should be the highest.
Choice (4)

25

You might also like